Chiusura forte e debole in \(L^2([0,1])\)

Messaggioda Delirium » 19/08/2016, 19:54

Esercizio. Sia \(K \subseteq \mathbb{R}\) un insieme chiuso e consideriamo l'insieme di funzioni \[X=\{f \in L^2([0,1]) \, : \, f(x) \in K \text{ per q.o. } x \in K \}. \]
1. Provare che \(X\) è chiuso in \(L^2([0,1])\) per la convergenza forte.
2. Sia ora \(K \subseteq \mathbb{R}\) un intervallo chiuso. Provare che \(X\) è chiuso per la convergenza debole di \(L^2([0,1])\).

Credo di avere una soluzione per 1, mentre non ho ancora provato a fare 2. Divertitevi.
Delirium
 

Re: Chiusura forte e debole in \(L^2([0,1])\)

Messaggioda Vincent46 » 20/08/2016, 10:58

Provo ma non sono sicuro perché teoria della misura è difficile :-D

Per beneficio personale mi appunto che:

Sia $A \subset \mathbb{R^n}$.
Si dice che una successione di funzioni $\{f_n\}$, con $f_n \in L^2(A)$, converge fortemente a una funzione $f \in L^2(A)$ se $f_n \to f$ in norma $L^2(A)$; ovvero,
$$\lim_{n \to \infty} \left( \int_A |f_n-f|^2 \right)^{\frac12}= 0 \ .$$
Si dice che una successione di funzioni $\{f_n\}$, con $f_n \in L^2(A)$, converge debolmente a una funzione $f \in L^2(A)$ se $\langle f_n, g \rangle \to \langle f, g \rangle $ per ogni $g \in L^2(A)$; ovvero,
$$\lim_{n \to \infty} \int_A f_ng \, dx = \int_A fg \ .$$

Testo nascosto, fai click qui per vederlo
1. Sia $f_n \in X$ convergente fortemente a $f \in L^2(0,1)$. Devo dimostrare che $f \in X$, ovvero che $f(x) \in K$ quasi ovunque. Per assurdo, supponiamo che esista un insieme $A \subseteq K$ di misura strettamente positiva tale che $f(A) \cap K = \emptyset$. Allora, per la proprietà di regolarità dall'interno della misura di Lebesgue, esiste un insieme compatto $B \subset A$ di misura strettamente positiva. Sia ora
$$ c = \text{dist}(K, f(B)) = \inf \{d(x,y):x \in K, y \in f(B)\} \, .$$
Poiché la funzione $d$ varia in $K \times B$, che è compatto, l' $\text{inf}$ è in realtà un $\min$; inoltre, poiché $K$ e $B$ sono disgiunti, $c$ è strettamente positivo.

EDIT: per completezza, segnalo che l'ultimo passaggio è sbagliato, in quanto $K$ è solo chiuso, ma non necessariamente compatto. D'altronde la dimostrazione può procedere ugualmente, in quanto vale il seguente risultato: se $U$ è un sottoinsieme compatto e $V$ è un sottoinsieme chiuso di uno spazio metrico, con $U \cap V = \emptyset$, allora $\text{dist}(U, V)>0$.

Segue che, per ogni $n$ intero positivo,
$$ \int_0^1 |f_n-f|^2 \geq \int_B |f_n-f|^2 \geq c^2 \mu(B) > 0 \, ,$$
e dunque il limite di $\int_0^1 |f_n-f|^2$ sarà a maggior ragione maggiore di $c^2 \mu(B)$, contro l'ipotesi.

2. Sia $f_n \in X$ convergente debolmente a $f \in L^2(0,1)$. Devo dimostrare che $f \in X$, ovvero che $f(x) \in K$ quasi ovunque. Per assurdo, come prima, supponiamo che esista un insieme $A \subseteq K$ di misura strettamente positiva tale che $f(A) \cap K = \emptyset$. Allora, per la proprietà di regolarità dall'interno della misura di Lebesgue, esiste un insieme compatto $B \subset A$ di misura strettamente positiva. Definiamo ora
$$B^{+} = \{x \in B : f(x) > y, \ \forall y \in K\} \qquad \text{ e } \qquad B^{-} = \{x \in B : f(x) < y, \ \forall y \in K\} \ .$$
Poiché $K$ è un intervallo, si ha che $B = B^+ \cup B^-$, e quindi almeno uno dei due sottoinsiemi deve avere misura strettamente positiva. Senza perdita di generalità, supponiamo che $\mu(B^+) > 0$. Come prima, definiamo
$$ \text{dist}(K, f(B^+)) = c > 0 \ .$$
Allora, applicando la definizione di convergenza debole con $g = \chi_B$ (la funzione indicatrice di B), si ottiene che, per ogni $n$ intero positivo,
$$\int_0^1 f_n \chi_B = \int_B f_n \leq \int_B (f-c) = \left ( \int_B f \right )- c\mu(B) \, ,$$
e, passando al limite, si viola ancora una volta l'ipotesi di convergenza.
Ultima modifica di Vincent46 il 20/08/2016, 19:49, modificato 4 volte in totale.
Vincent46
Average Member
Average Member
 
Messaggio: 131 di 523
Iscritto il: 26/01/2014, 17:27

Re: Chiusura forte e debole in \(L^2([0,1])\)

Messaggioda Delirium » 20/08/2016, 15:44

@Vincent: ma \(A\) non dovrebbe essere sottoinsieme di \(K\)? Perché la condizione di appartenenza ad \(X\) è la "\(K\)-invarianza", cioè \(f \in X\) sse (oltre alla sommabilità) \(f(x) \in K\) per quasi ogni \(x \in K\)... in tal caso si avrebbe \(c=0\).

Edit. Forse il testo del problema è anche un po' ambiguo. \(K \) è detto essere sottoinsieme di \(\mathbb{R}\), ma potrebbe essere disgiunto da \([0,1]\) (se \(K\) fosse un sottoinsieme di \([0,1]\) sarebbe facile, la convergenza in \(L^p\) implica la convergenza q.o. di una sottosuccessione). Nel caso di \(K\) disgiunto da \([0,1]\) interpreto \(X\) come l'insieme delle funzioni \(f : \mathbb{R} \to \mathbb{R}\) tali che \(f \in L^2([0,1])\) e \(f(x) \in K \) per quasi ogni \(x \in K \). Dimmi se questa cosa ti torna, prima che (io) scriva fesserie dettate dalla mala interpretazione del testo...
Delirium
 

Messaggioda j18eos » 20/08/2016, 16:36

@Vincent Mi permetto di farti notare che \(\displaystyle K\) non è compatto per ipotesi!
Ipocrisìa e omofobìa,
fuori da casa mia!

Semplicemente Armando. ;)
Avatar utente
j18eos
Moderatore
Moderatore
 
Messaggio: 5785 di 13405
Iscritto il: 12/06/2010, 15:27
Località: Napoli, Trieste, ed ogni tanto a Roma ^_^

Re: Chiusura forte e debole in \(L^2([0,1])\)

Messaggioda Vincent46 » 20/08/2016, 17:51

Uh, avete ragione. Io in effetti avevo inteso che $K \subseteq [0, 1]$, il che risolverebbe anche la questione della compattezza ($K$ risulterebbe chiuso e limitato in $\mathbb{R$}. Lo spazio $L^p(A)$ non è definito come lo spazio delle funzioni $f : A \to \mathbb{R}$ (oppure $f : A \to \mathbb{C}$) tali che $|f|_{L^p} < \infty$?
Se supponiamo $K \subseteq [0, 1]$: $A$ risulterebbe in effetti un sottoinsieme di $K$, ma perché si dovrebbe avere $c=0$?

EDIT: ho anche pensato al caso $K \subseteq \mathbb{R}$, $f_n : [0, 1] \to \mathbb{R}$ (quindi $f$ resta definita solo su $[0,1]$). Allora, non si può ripetere la dimostrazione che ho dato sopra così com'è, perché $K$ non è più compatto. Tuttavia sussiste un risultato che permette di salvarla, ossia: se $U$ è un insieme compatto e $V$ è un insieme chiuso in uno spazio metrico e $U, V$ sono disgiunti, allora $\text{dist}(U, V) > 0$. Così, a meno di erroracci, dovrebbe andare bene.
Vincent46
Average Member
Average Member
 
Messaggio: 133 di 523
Iscritto il: 26/01/2014, 17:27

Re: Chiusura forte e debole in \(L^2([0,1])\)

Messaggioda Delirium » 20/08/2016, 18:27

Vincent46 ha scritto:[...] Lo spazio $L^p(A)$ non è definito come lo spazio delle funzioni $f : A \to \mathbb{R}$ (oppure $f : A \to \mathbb{C}$) tali che $|f|_{L^p} < \infty$? [...]

Sì, ed è per quello che sono perplesso.

Vincent46 ha scritto:[...] Se supponiamo $K \subset [0, 1]$: $A$ risulterebbe in effetti un sottoinsieme di $K$, ma perché si dovrebbe avere $c=0$?

Perché \(B \subset A \subset K\), e \(c\) è la distanza tra \(B\) e \(K\), ma la distanza tra un insieme ed un suo sottoinsieme proprio è \(=0\)... o vedo male? Inoltre tu supponi \(A \subset K\) di misura positiva; ma cosa succede se \(\mu(K)=0\)? Questa considerazione ulteriore mi fa pensare che il testo potrebbe essere formulato in maniera migliore.

Poi se supponiamo \(K \subset [0,1]\) mi sembra che si possa semplicemente dire che: \(\|f_n - f \|_{L^2([0,1])} \to 0 \) implica, a meno di sottosuccessioni, \(f_n (x) \to f(x) \) quasi ovunque in \([0,1]\). Posto poi \[A_n = \{x \in K \, : \, f_n (x) \notin K \} \]si ha che, per ipotesi, \(\mu (A_n) = 0 \) per ogni \(n \in \mathbb{N}\); e inoltre \( \mu \left( \cup_{n \in \mathbb{N}} A_n \right) = 0 \). Pertanto per ogni \(x \in K \setminus \cup_{n \in \mathbb{N}} A_n \) si ha che \(f_n (x) \in K\), e questo per ogni \(n \in \mathbb{N}\). Siccome \(K\) è chiuso \(f(x) \in K\) necessariamente.
Delirium
 

Re: Chiusura forte e debole in \(L^2([0,1])\)

Messaggioda Vincent46 » 20/08/2016, 19:10

Perché \(B \subset A \subset K\), e \(c\) è la distanza tra \(B\) e \(K\), ma la distanza tra un insieme ed un suo sottoinsieme proprio è \(=0\)... o vedo male?

hai ragionissima; non intendevo scrivere $\text{dist}(B, K)$, bensì $\text{dist}(f(B), K)$. Perdonami, mi è sfuggito: ora modifico.

Inoltre tu supponi \(A \subset K\) di misura positiva; ma cosa succede se \(\mu(K)=0\)?

In quel caso è la tesi sarebbe verificata banalmente, no? Infatti sarebbe certamente vero che $f(x) \in K$ per quasi ogni $x \in K$, in quanto quel "quasi ogni" permette di escludere l'intero $K$.

Riguardo alla tua dimostrazione: molto semplice ed elegante. Non ricordavo più che la convergenza in norma $L^p$ implicasse la convergenza q.o. di una sottosuccessione.

In ogni caso, mi sembra che le dimostrazioni date finora si possano ripetere senza problemi sostituendo l'insieme $X$ dell'esercizio con l'insieme
$$X=\{f \in L^2([0,1]):f(x) \in K \text{ per q.o. } x \in[0,1]\} \, ,$$
quindi con una tesi più forte (sempre supponendo che le funzioni appartenenti $X$ siano definite solo su $[0,1]$).
Vincent46
Average Member
Average Member
 
Messaggio: 134 di 523
Iscritto il: 26/01/2014, 17:27

Re: Chiusura forte e debole in \(L^2([0,1])\)

Messaggioda Delirium » 20/08/2016, 20:04

Vincent46 ha scritto:
Perché \(B \subset A \subset K\), e \(c\) è la distanza tra \(B\) e \(K\), ma la distanza tra un insieme ed un suo sottoinsieme proprio è \(=0\)... o vedo male?

hai ragionissima; non intendevo scrivere $\text{dist}(B, K)$, bensì $\text{dist}(f(B), K)$. Perdonami, mi è sfuggito: ora modifico.

Ok, no prob. Però continuo ad essere perplesso (ho letto anche il tuo edit dentro lo spoiler). Dici che, in uno spazio metrico, un compatto ed un chiuso, se disgiunti, hanno distanza \(>0\); e sono d'accordo. Il problema è che tu, ora, stai calcolando la distanza tra \(K\) (chiuso) e \(f(B)\) (???), che senza ulteriori ipotesi su \(f\) potrebbe non essere né chiuso né compatto... vedo male?

Vincent46 ha scritto:[...] In quel caso è la tesi sarebbe verificata banalmente, no? Infatti sarebbe certamente vero che $f(x) \in K$ per quasi ogni $x \in K$, in quanto quel "quasi ogni" permette di escludere l'intero $K$. [...]

Non so, qui è una questione di terminologia - non so se abbia senso, terminologicamente parlando, discutere di proprietà valide "quasi ovunque" in insiemi di misura nulla. Ma è probabile che tu abbia ragione.

Che bello questo topic!
Delirium
 

Re: Chiusura forte e debole in \(L^2([0,1])\)

Messaggioda Vincent46 » 20/08/2016, 20:35

Hai ragione, che stupido :| . Allora non so come rimediare. Domani ci ripenso.
Vincent46
Average Member
Average Member
 
Messaggio: 135 di 523
Iscritto il: 26/01/2014, 17:27

Re: Chiusura forte e debole in \(L^2([0,1])\)

Messaggioda ViciousGoblin » 20/12/2016, 14:44

Ho visto questo thread girando (dopo molto tempo) per il forum. Avete risolto?
Per il primo quesito si dovrebbe usare il fatto che ogni successione convergente in $L^2$ ammette una sottosuccessione che converge quasi ovunque.
You are in a comfortable tunnel like hall.
To the east there is a round green door.
>OPEN DOOR
>GO EAST
静かに時の傷に苦しむ
群れを組んでわ飛ばない鷹
Avatar utente
ViciousGoblin
Advanced Member
Advanced Member
 
Messaggio: 1588 di 2036
Iscritto il: 09/03/2008, 17:38
Località: Pisa

Prossimo

Torna a Pensare un po' di più

Chi c’è in linea

Visitano il forum: Nessuno e 1 ospite